LSAT and Law School Admissions Forum

Get expert LSAT preparation and law school admissions advice from PowerScore Test Preparation.

 MeliXi
  • Posts: 19
  • Joined: Dec 12, 2020
|
#83444
Hello;

I had trouble with this question, narrowing it down to C and D, but ultimately chose D because I felt like C was a bit problematic.
The conclusion is that some subsidiaries will be sold, but I feel like if we add C in, then workers not going on strike is more of a conclusion than the subsidiaries being sold.
We see that workers will call off their strike only if management agrees to raise wages. So to me, the step by step order seems to be this:
Bell has to sell subsidiaries in order to pay for the increase in workers' wages --> the subsidiaries will be sold --> there's now money to pay for the increase in wages --> now that the workers have their demands satisfied, they won't strike.

I feel like choice C is arguing: because the workers are not going on strike, Bell is selling some subsidiaries to pay for wage increases.
using the Justify Formula:
(Bell has to sell subsidiaries to pay for wage increases) + (the workers will not be going on strike) = (some of Bell's subsidiaries will be sold.) this doesn't make any sense to me!
this makes more sense:
(Bell has to sell subsidiaries to pay for wage increases) + (some of Bell's subsidiaries will be sold) = the workers will not be going on strike.
 Adam Tyson
PowerScore Staff
  • PowerScore Staff
  • Posts: 5153
  • Joined: Apr 14, 2011
|
#83752
It looks to me like you are thinking about these relationships chronologically rather than conditionally, MeliXi. Check out the conditional chain in the official explanation at the top of this thread, and you will see that the workers not going on strike is sufficient to prove that wages are being increased, which is sufficient to prove that subsidiaries will be sold. Conditional relationships aren't necessarily chronological - I think you're right that the order of events would be to sell subsidiaries, then use the money to raise wages, and that would have the effect of preventing a strike. But then again, maybe the order is that the company tells the workers what they plan to do, and the workers agree not to strike, after which the company raises wages and then sells subsidiaries in order to pay for their action.

Don't focus on the chronology! How do we prove (justify) a necessary condition will occur? We say that something sufficient for it will occur. That's all this question is asking you to do - state that the sufficient condition will occur, thereby proving that the necessary condition will occur.

And don't forget, the stem is telling you to accept the answers as true ("if which one of the following is assumed?"), so we shouldn't be thinking of the answer choices as things that need proof. They are facts, and the only question is what those facts prove. Answer C, if taken as fact, proves the conclusion of the argument. Answer D does not.
User avatar
 elite097
  • Posts: 18
  • Joined: Apr 09, 2023
|
#102387
The reason for rejecting E is not clear at all. They’ll have to pay wages if workers don’t want benefits.
 Rachael Wilkenfeld
PowerScore Staff
  • PowerScore Staff
  • Posts: 1358
  • Joined: Dec 15, 2011
|
#102441
Answer choice (E) leaves open another possibility, Elite. The workers could just strike. We need something that proves that the company would sell the subsidiaries. Knowing that the workers want cash and not benefits doesn't prove that they will be sold; it just tells us more about the workers' views.

Since this argument is conditional, we have an easy way to prove something. Look for the conditional reasoning---we can use it to prove the conclusion follows. When we have the sufficient, the necessary MUST follow. In Justify questions, we can use this reasoning to our advantage. Just find a statement that is sufficient to know that some of the subsidiaries will be sold.

There are actually two different things that are sufficient to know that the subsidiaries will be sold. 1) We could know that the workers' wages were increased or 2) we could know that the workers won't strike. Either way, we prove that the subsidiaries must be sold. I diagrammed these relationships a few posts up if you want to see what the conditional chains are.

Hope that helps!

Get the most out of your LSAT Prep Plus subscription.

Analyze and track your performance with our Testing and Analytics Package.